Almost uniform convergence implies convergence in measureAlmost uniform convergence of $f_n(x) = x^n$ on the interval $[0, 1]$?Uniform convergence and pointwise convergence, understandingQuantifiers in the definition of uniform convergenceDominated a.e. convergence implies almost uniform convergenceEquivalent definition of almost uniform convergenceHow to show that the given sequence of functions converges to $f$ almost everywhere, almost uniformly and in measure?Showing that the almost uniform limit of functions with bounded $L_infty$ norms is in $L_infty$Convergence in measure and almost everywhere proof

Justifying the use of directed energy weapons

How to respectfully refuse to assist co-workers with IT issues?

What is a CirKle Word™?

Is "The life is beautiful" incorrect or just very non-idiomatic?

Avoiding racist tropes in fantasy

Compelling story with the world as a villain

Why does The Ancient One think differently about Doctor Strange in Endgame than the film Doctor Strange?

Are there languages that inflect adverbs for gender

Was there ever a treaty between 2 entities with significantly different translations to the detriment of one party?

How do you harvest carrots in creative mode?

Almost uniform convergence implies convergence in measure

Prove your innocence

I don't have the theoretical background in my PhD topic. I can't justify getting the degree

“T” in subscript in formulas

Are modern clipless shoes and pedals that much better than toe clips and straps?

Numbers Decrease while Letters Increase

Immutable builder and updater

How should I face my manager if I make a mistake because a senior coworker explained something incorrectly to me?

Why do gliders have bungee cords in the control systems and what do they do? Are they on all control surfaces? What about ultralights?

Did anyone try to find the little box that held Professor Moriarty and his wife after the Enterprise D crashed?

Round towards zero

Strange-looking FM transmitter circuit

How to determine car loan length as a function of how long I plan to keep a car

Non-visual Computers - thoughts?



Almost uniform convergence implies convergence in measure


Almost uniform convergence of $f_n(x) = x^n$ on the interval $[0, 1]$?Uniform convergence and pointwise convergence, understandingQuantifiers in the definition of uniform convergenceDominated a.e. convergence implies almost uniform convergenceEquivalent definition of almost uniform convergenceHow to show that the given sequence of functions converges to $f$ almost everywhere, almost uniformly and in measure?Showing that the almost uniform limit of functions with bounded $L_infty$ norms is in $L_infty$Convergence in measure and almost everywhere proof






.everyoneloves__top-leaderboard:empty,.everyoneloves__mid-leaderboard:empty,.everyoneloves__bot-mid-leaderboard:empty margin-bottom:0;








1












$begingroup$


Let $(A,mathcalF,mu)$ be finite measure space and $f_n$ a sequence of finite real measurable functions so that $f_nrightarrow f$ a.e. We say $f_nrightarrow f$ almost uniformly if $epsilon>0$, there is $Esubseteq A$ such that $f_n rightarrow f$ uniformly on $E^c$ and $mu(E)<epsilon$.



I want to show that $f_nrightarrow f$ almost uniformly implies convergence in $mu$. For this, suppose not. Then
$$exists eta,epsilon>0:forall Nin mathbbN:exists n>N:mu(mid f_n-fmidgeqepsilon)geq eta, $$
i.e., for infinitely many points $nin mathbbN$. From the definition of almost uniform convergence, $exists E:mu(E)<eta$ and $f_nrightarrow f$ uniformly on $E^c$. Contradiction.



Question



It seems intuitive to me. But how to deduce this contradiction precisely?
I know that if $xin E$, then it must satify the negation of uniform convergence which is
$$exists epsilon>0:forall Nin mathbbN:exists n>N:mid f_n-fmidgeqepsilon.$$
Now, $x$ may not be in $f_n text does not converge in measure to f $ if $mu(mid f_n(x)-f(x)midgeqepsilon)<eta$. So I conclude that $$f_n text does not converge in measure to f subseteq E$$ implying that $eta>mu(E)geq eta$; a contradiction.



My argument seems right but also very inefficient. How could you express this idea as clean as possible?



Thanks!










share|cite|improve this question











$endgroup$




















    1












    $begingroup$


    Let $(A,mathcalF,mu)$ be finite measure space and $f_n$ a sequence of finite real measurable functions so that $f_nrightarrow f$ a.e. We say $f_nrightarrow f$ almost uniformly if $epsilon>0$, there is $Esubseteq A$ such that $f_n rightarrow f$ uniformly on $E^c$ and $mu(E)<epsilon$.



    I want to show that $f_nrightarrow f$ almost uniformly implies convergence in $mu$. For this, suppose not. Then
    $$exists eta,epsilon>0:forall Nin mathbbN:exists n>N:mu(mid f_n-fmidgeqepsilon)geq eta, $$
    i.e., for infinitely many points $nin mathbbN$. From the definition of almost uniform convergence, $exists E:mu(E)<eta$ and $f_nrightarrow f$ uniformly on $E^c$. Contradiction.



    Question



    It seems intuitive to me. But how to deduce this contradiction precisely?
    I know that if $xin E$, then it must satify the negation of uniform convergence which is
    $$exists epsilon>0:forall Nin mathbbN:exists n>N:mid f_n-fmidgeqepsilon.$$
    Now, $x$ may not be in $f_n text does not converge in measure to f $ if $mu(mid f_n(x)-f(x)midgeqepsilon)<eta$. So I conclude that $$f_n text does not converge in measure to f subseteq E$$ implying that $eta>mu(E)geq eta$; a contradiction.



    My argument seems right but also very inefficient. How could you express this idea as clean as possible?



    Thanks!










    share|cite|improve this question











    $endgroup$
















      1












      1








      1





      $begingroup$


      Let $(A,mathcalF,mu)$ be finite measure space and $f_n$ a sequence of finite real measurable functions so that $f_nrightarrow f$ a.e. We say $f_nrightarrow f$ almost uniformly if $epsilon>0$, there is $Esubseteq A$ such that $f_n rightarrow f$ uniformly on $E^c$ and $mu(E)<epsilon$.



      I want to show that $f_nrightarrow f$ almost uniformly implies convergence in $mu$. For this, suppose not. Then
      $$exists eta,epsilon>0:forall Nin mathbbN:exists n>N:mu(mid f_n-fmidgeqepsilon)geq eta, $$
      i.e., for infinitely many points $nin mathbbN$. From the definition of almost uniform convergence, $exists E:mu(E)<eta$ and $f_nrightarrow f$ uniformly on $E^c$. Contradiction.



      Question



      It seems intuitive to me. But how to deduce this contradiction precisely?
      I know that if $xin E$, then it must satify the negation of uniform convergence which is
      $$exists epsilon>0:forall Nin mathbbN:exists n>N:mid f_n-fmidgeqepsilon.$$
      Now, $x$ may not be in $f_n text does not converge in measure to f $ if $mu(mid f_n(x)-f(x)midgeqepsilon)<eta$. So I conclude that $$f_n text does not converge in measure to f subseteq E$$ implying that $eta>mu(E)geq eta$; a contradiction.



      My argument seems right but also very inefficient. How could you express this idea as clean as possible?



      Thanks!










      share|cite|improve this question











      $endgroup$




      Let $(A,mathcalF,mu)$ be finite measure space and $f_n$ a sequence of finite real measurable functions so that $f_nrightarrow f$ a.e. We say $f_nrightarrow f$ almost uniformly if $epsilon>0$, there is $Esubseteq A$ such that $f_n rightarrow f$ uniformly on $E^c$ and $mu(E)<epsilon$.



      I want to show that $f_nrightarrow f$ almost uniformly implies convergence in $mu$. For this, suppose not. Then
      $$exists eta,epsilon>0:forall Nin mathbbN:exists n>N:mu(mid f_n-fmidgeqepsilon)geq eta, $$
      i.e., for infinitely many points $nin mathbbN$. From the definition of almost uniform convergence, $exists E:mu(E)<eta$ and $f_nrightarrow f$ uniformly on $E^c$. Contradiction.



      Question



      It seems intuitive to me. But how to deduce this contradiction precisely?
      I know that if $xin E$, then it must satify the negation of uniform convergence which is
      $$exists epsilon>0:forall Nin mathbbN:exists n>N:mid f_n-fmidgeqepsilon.$$
      Now, $x$ may not be in $f_n text does not converge in measure to f $ if $mu(mid f_n(x)-f(x)midgeqepsilon)<eta$. So I conclude that $$f_n text does not converge in measure to f subseteq E$$ implying that $eta>mu(E)geq eta$; a contradiction.



      My argument seems right but also very inefficient. How could you express this idea as clean as possible?



      Thanks!







      real-analysis measure-theory proof-writing






      share|cite|improve this question















      share|cite|improve this question













      share|cite|improve this question




      share|cite|improve this question








      edited 7 hours ago









      Gabriel Romon

      19.2k5 gold badges37 silver badges88 bronze badges




      19.2k5 gold badges37 silver badges88 bronze badges










      asked 8 hours ago









      DanmatDanmat

      41610 bronze badges




      41610 bronze badges























          2 Answers
          2






          active

          oldest

          votes


















          4













          $begingroup$

          Here's a way without going for contradiction: Let $epsilon >0$ be fixed and consider some $delta >0$.



          By almost uniform convergence, there exists some $E$ with $mu(E)leq delta$ and some $N$ such that $ngeq Nimplies forall xin E^c, |f_n(x)-f(x)|< epsilon$.



          For $ngeq N$, note the inclusion $(|f_n-f|geq epsilon) subset E$, hence $mu((|f_n-f|geq epsilon))leq mu(E)leq delta$.



          Hence $forall delta>0, exists N, ngeq N implies mu((|f_n-f|geq epsilon))leq delta$. Thus $ mu((|f_n-f|geq epsilon)) to 0$.






          share|cite|improve this answer











          $endgroup$














          • $begingroup$
            Thanks! The inclusion $mid f_n-fmidgeq epsilonsubset E$ that I don't understand. What is this infinity-norm that you wrote?
            $endgroup$
            – Danmat
            8 hours ago










          • $begingroup$
            @Danmat I removed the infinity norm (which denotes the sup-norm), do you understand now ?
            $endgroup$
            – Gabriel Romon
            8 hours ago










          • $begingroup$
            It has something to do with the fact that uniform convergence implies pointwise convergence? And then the complement of the latter is contained in the complement of the former.
            $endgroup$
            – Danmat
            7 hours ago











          • $begingroup$
            @Danmat I just wrote the definition of uniform convergence on $E^c$. Pointwise convergence is not used in the proof.
            $endgroup$
            – Gabriel Romon
            7 hours ago










          • $begingroup$
            But, $E$ is constituted of elements in $A$ such that $f_n rightarrow f$ does not converge uniformly, right?
            $endgroup$
            – Danmat
            7 hours ago



















          2













          $begingroup$

          This is proved more succinctly by a direct proof.



          Given $varepsilon > 0,$ let $E$ be as in the definition of almost-uniform convergence. Then there is some $N$ such that $n geq N$ implies
          $$mu(E^c cap geqvarepsilon) = 0$$
          What does this imply for $mu(|f_n-f|geq varepsilon)?$






          share|cite|improve this answer









          $endgroup$














          • $begingroup$
            How to conclude that $mid f_n-fmid subseteq E$?
            $endgroup$
            – Danmat
            7 hours ago










          • $begingroup$
            @Danmat Forget about $E$ for a second. On $E^c,$ the functions converge uniformly. This means that there will be some $N$ such that $n geq N$ implies $|f_n - f| < varepsilon.$ In particular, on $E^c,$ $$n geq N implies geq varepsilon = emptyset.$$ This directly means $$E^c cap geq varepsilon = emptyset.$$ If you want, then you can use the property of sets that $$A^c cap B = emptyset iff B subseteq A$$
            $endgroup$
            – Brian Moehring
            7 hours ago











          • $begingroup$
            Very nice! Now, it's all clear to me!
            $endgroup$
            – Danmat
            7 hours ago













          Your Answer








          StackExchange.ready(function()
          var channelOptions =
          tags: "".split(" "),
          id: "69"
          ;
          initTagRenderer("".split(" "), "".split(" "), channelOptions);

          StackExchange.using("externalEditor", function()
          // Have to fire editor after snippets, if snippets enabled
          if (StackExchange.settings.snippets.snippetsEnabled)
          StackExchange.using("snippets", function()
          createEditor();
          );

          else
          createEditor();

          );

          function createEditor()
          StackExchange.prepareEditor(
          heartbeatType: 'answer',
          autoActivateHeartbeat: false,
          convertImagesToLinks: true,
          noModals: true,
          showLowRepImageUploadWarning: true,
          reputationToPostImages: 10,
          bindNavPrevention: true,
          postfix: "",
          imageUploader:
          brandingHtml: "Powered by u003ca class="icon-imgur-white" href="https://imgur.com/"u003eu003c/au003e",
          contentPolicyHtml: "User contributions licensed under u003ca href="https://creativecommons.org/licenses/by-sa/3.0/"u003ecc by-sa 3.0 with attribution requiredu003c/au003e u003ca href="https://stackoverflow.com/legal/content-policy"u003e(content policy)u003c/au003e",
          allowUrls: true
          ,
          noCode: true, onDemand: true,
          discardSelector: ".discard-answer"
          ,immediatelyShowMarkdownHelp:true
          );



          );













          draft saved

          draft discarded


















          StackExchange.ready(
          function ()
          StackExchange.openid.initPostLogin('.new-post-login', 'https%3a%2f%2fmath.stackexchange.com%2fquestions%2f3332265%2falmost-uniform-convergence-implies-convergence-in-measure%23new-answer', 'question_page');

          );

          Post as a guest















          Required, but never shown

























          2 Answers
          2






          active

          oldest

          votes








          2 Answers
          2






          active

          oldest

          votes









          active

          oldest

          votes






          active

          oldest

          votes









          4













          $begingroup$

          Here's a way without going for contradiction: Let $epsilon >0$ be fixed and consider some $delta >0$.



          By almost uniform convergence, there exists some $E$ with $mu(E)leq delta$ and some $N$ such that $ngeq Nimplies forall xin E^c, |f_n(x)-f(x)|< epsilon$.



          For $ngeq N$, note the inclusion $(|f_n-f|geq epsilon) subset E$, hence $mu((|f_n-f|geq epsilon))leq mu(E)leq delta$.



          Hence $forall delta>0, exists N, ngeq N implies mu((|f_n-f|geq epsilon))leq delta$. Thus $ mu((|f_n-f|geq epsilon)) to 0$.






          share|cite|improve this answer











          $endgroup$














          • $begingroup$
            Thanks! The inclusion $mid f_n-fmidgeq epsilonsubset E$ that I don't understand. What is this infinity-norm that you wrote?
            $endgroup$
            – Danmat
            8 hours ago










          • $begingroup$
            @Danmat I removed the infinity norm (which denotes the sup-norm), do you understand now ?
            $endgroup$
            – Gabriel Romon
            8 hours ago










          • $begingroup$
            It has something to do with the fact that uniform convergence implies pointwise convergence? And then the complement of the latter is contained in the complement of the former.
            $endgroup$
            – Danmat
            7 hours ago











          • $begingroup$
            @Danmat I just wrote the definition of uniform convergence on $E^c$. Pointwise convergence is not used in the proof.
            $endgroup$
            – Gabriel Romon
            7 hours ago










          • $begingroup$
            But, $E$ is constituted of elements in $A$ such that $f_n rightarrow f$ does not converge uniformly, right?
            $endgroup$
            – Danmat
            7 hours ago
















          4













          $begingroup$

          Here's a way without going for contradiction: Let $epsilon >0$ be fixed and consider some $delta >0$.



          By almost uniform convergence, there exists some $E$ with $mu(E)leq delta$ and some $N$ such that $ngeq Nimplies forall xin E^c, |f_n(x)-f(x)|< epsilon$.



          For $ngeq N$, note the inclusion $(|f_n-f|geq epsilon) subset E$, hence $mu((|f_n-f|geq epsilon))leq mu(E)leq delta$.



          Hence $forall delta>0, exists N, ngeq N implies mu((|f_n-f|geq epsilon))leq delta$. Thus $ mu((|f_n-f|geq epsilon)) to 0$.






          share|cite|improve this answer











          $endgroup$














          • $begingroup$
            Thanks! The inclusion $mid f_n-fmidgeq epsilonsubset E$ that I don't understand. What is this infinity-norm that you wrote?
            $endgroup$
            – Danmat
            8 hours ago










          • $begingroup$
            @Danmat I removed the infinity norm (which denotes the sup-norm), do you understand now ?
            $endgroup$
            – Gabriel Romon
            8 hours ago










          • $begingroup$
            It has something to do with the fact that uniform convergence implies pointwise convergence? And then the complement of the latter is contained in the complement of the former.
            $endgroup$
            – Danmat
            7 hours ago











          • $begingroup$
            @Danmat I just wrote the definition of uniform convergence on $E^c$. Pointwise convergence is not used in the proof.
            $endgroup$
            – Gabriel Romon
            7 hours ago










          • $begingroup$
            But, $E$ is constituted of elements in $A$ such that $f_n rightarrow f$ does not converge uniformly, right?
            $endgroup$
            – Danmat
            7 hours ago














          4














          4










          4







          $begingroup$

          Here's a way without going for contradiction: Let $epsilon >0$ be fixed and consider some $delta >0$.



          By almost uniform convergence, there exists some $E$ with $mu(E)leq delta$ and some $N$ such that $ngeq Nimplies forall xin E^c, |f_n(x)-f(x)|< epsilon$.



          For $ngeq N$, note the inclusion $(|f_n-f|geq epsilon) subset E$, hence $mu((|f_n-f|geq epsilon))leq mu(E)leq delta$.



          Hence $forall delta>0, exists N, ngeq N implies mu((|f_n-f|geq epsilon))leq delta$. Thus $ mu((|f_n-f|geq epsilon)) to 0$.






          share|cite|improve this answer











          $endgroup$



          Here's a way without going for contradiction: Let $epsilon >0$ be fixed and consider some $delta >0$.



          By almost uniform convergence, there exists some $E$ with $mu(E)leq delta$ and some $N$ such that $ngeq Nimplies forall xin E^c, |f_n(x)-f(x)|< epsilon$.



          For $ngeq N$, note the inclusion $(|f_n-f|geq epsilon) subset E$, hence $mu((|f_n-f|geq epsilon))leq mu(E)leq delta$.



          Hence $forall delta>0, exists N, ngeq N implies mu((|f_n-f|geq epsilon))leq delta$. Thus $ mu((|f_n-f|geq epsilon)) to 0$.







          share|cite|improve this answer














          share|cite|improve this answer



          share|cite|improve this answer








          edited 8 hours ago

























          answered 8 hours ago









          Gabriel RomonGabriel Romon

          19.2k5 gold badges37 silver badges88 bronze badges




          19.2k5 gold badges37 silver badges88 bronze badges














          • $begingroup$
            Thanks! The inclusion $mid f_n-fmidgeq epsilonsubset E$ that I don't understand. What is this infinity-norm that you wrote?
            $endgroup$
            – Danmat
            8 hours ago










          • $begingroup$
            @Danmat I removed the infinity norm (which denotes the sup-norm), do you understand now ?
            $endgroup$
            – Gabriel Romon
            8 hours ago










          • $begingroup$
            It has something to do with the fact that uniform convergence implies pointwise convergence? And then the complement of the latter is contained in the complement of the former.
            $endgroup$
            – Danmat
            7 hours ago











          • $begingroup$
            @Danmat I just wrote the definition of uniform convergence on $E^c$. Pointwise convergence is not used in the proof.
            $endgroup$
            – Gabriel Romon
            7 hours ago










          • $begingroup$
            But, $E$ is constituted of elements in $A$ such that $f_n rightarrow f$ does not converge uniformly, right?
            $endgroup$
            – Danmat
            7 hours ago

















          • $begingroup$
            Thanks! The inclusion $mid f_n-fmidgeq epsilonsubset E$ that I don't understand. What is this infinity-norm that you wrote?
            $endgroup$
            – Danmat
            8 hours ago










          • $begingroup$
            @Danmat I removed the infinity norm (which denotes the sup-norm), do you understand now ?
            $endgroup$
            – Gabriel Romon
            8 hours ago










          • $begingroup$
            It has something to do with the fact that uniform convergence implies pointwise convergence? And then the complement of the latter is contained in the complement of the former.
            $endgroup$
            – Danmat
            7 hours ago











          • $begingroup$
            @Danmat I just wrote the definition of uniform convergence on $E^c$. Pointwise convergence is not used in the proof.
            $endgroup$
            – Gabriel Romon
            7 hours ago










          • $begingroup$
            But, $E$ is constituted of elements in $A$ such that $f_n rightarrow f$ does not converge uniformly, right?
            $endgroup$
            – Danmat
            7 hours ago
















          $begingroup$
          Thanks! The inclusion $mid f_n-fmidgeq epsilonsubset E$ that I don't understand. What is this infinity-norm that you wrote?
          $endgroup$
          – Danmat
          8 hours ago




          $begingroup$
          Thanks! The inclusion $mid f_n-fmidgeq epsilonsubset E$ that I don't understand. What is this infinity-norm that you wrote?
          $endgroup$
          – Danmat
          8 hours ago












          $begingroup$
          @Danmat I removed the infinity norm (which denotes the sup-norm), do you understand now ?
          $endgroup$
          – Gabriel Romon
          8 hours ago




          $begingroup$
          @Danmat I removed the infinity norm (which denotes the sup-norm), do you understand now ?
          $endgroup$
          – Gabriel Romon
          8 hours ago












          $begingroup$
          It has something to do with the fact that uniform convergence implies pointwise convergence? And then the complement of the latter is contained in the complement of the former.
          $endgroup$
          – Danmat
          7 hours ago





          $begingroup$
          It has something to do with the fact that uniform convergence implies pointwise convergence? And then the complement of the latter is contained in the complement of the former.
          $endgroup$
          – Danmat
          7 hours ago













          $begingroup$
          @Danmat I just wrote the definition of uniform convergence on $E^c$. Pointwise convergence is not used in the proof.
          $endgroup$
          – Gabriel Romon
          7 hours ago




          $begingroup$
          @Danmat I just wrote the definition of uniform convergence on $E^c$. Pointwise convergence is not used in the proof.
          $endgroup$
          – Gabriel Romon
          7 hours ago












          $begingroup$
          But, $E$ is constituted of elements in $A$ such that $f_n rightarrow f$ does not converge uniformly, right?
          $endgroup$
          – Danmat
          7 hours ago





          $begingroup$
          But, $E$ is constituted of elements in $A$ such that $f_n rightarrow f$ does not converge uniformly, right?
          $endgroup$
          – Danmat
          7 hours ago














          2













          $begingroup$

          This is proved more succinctly by a direct proof.



          Given $varepsilon > 0,$ let $E$ be as in the definition of almost-uniform convergence. Then there is some $N$ such that $n geq N$ implies
          $$mu(E^c cap geqvarepsilon) = 0$$
          What does this imply for $mu(|f_n-f|geq varepsilon)?$






          share|cite|improve this answer









          $endgroup$














          • $begingroup$
            How to conclude that $mid f_n-fmid subseteq E$?
            $endgroup$
            – Danmat
            7 hours ago










          • $begingroup$
            @Danmat Forget about $E$ for a second. On $E^c,$ the functions converge uniformly. This means that there will be some $N$ such that $n geq N$ implies $|f_n - f| < varepsilon.$ In particular, on $E^c,$ $$n geq N implies geq varepsilon = emptyset.$$ This directly means $$E^c cap geq varepsilon = emptyset.$$ If you want, then you can use the property of sets that $$A^c cap B = emptyset iff B subseteq A$$
            $endgroup$
            – Brian Moehring
            7 hours ago











          • $begingroup$
            Very nice! Now, it's all clear to me!
            $endgroup$
            – Danmat
            7 hours ago















          2













          $begingroup$

          This is proved more succinctly by a direct proof.



          Given $varepsilon > 0,$ let $E$ be as in the definition of almost-uniform convergence. Then there is some $N$ such that $n geq N$ implies
          $$mu(E^c cap geqvarepsilon) = 0$$
          What does this imply for $mu(|f_n-f|geq varepsilon)?$






          share|cite|improve this answer









          $endgroup$














          • $begingroup$
            How to conclude that $mid f_n-fmid subseteq E$?
            $endgroup$
            – Danmat
            7 hours ago










          • $begingroup$
            @Danmat Forget about $E$ for a second. On $E^c,$ the functions converge uniformly. This means that there will be some $N$ such that $n geq N$ implies $|f_n - f| < varepsilon.$ In particular, on $E^c,$ $$n geq N implies geq varepsilon = emptyset.$$ This directly means $$E^c cap geq varepsilon = emptyset.$$ If you want, then you can use the property of sets that $$A^c cap B = emptyset iff B subseteq A$$
            $endgroup$
            – Brian Moehring
            7 hours ago











          • $begingroup$
            Very nice! Now, it's all clear to me!
            $endgroup$
            – Danmat
            7 hours ago













          2














          2










          2







          $begingroup$

          This is proved more succinctly by a direct proof.



          Given $varepsilon > 0,$ let $E$ be as in the definition of almost-uniform convergence. Then there is some $N$ such that $n geq N$ implies
          $$mu(E^c cap geqvarepsilon) = 0$$
          What does this imply for $mu(|f_n-f|geq varepsilon)?$






          share|cite|improve this answer









          $endgroup$



          This is proved more succinctly by a direct proof.



          Given $varepsilon > 0,$ let $E$ be as in the definition of almost-uniform convergence. Then there is some $N$ such that $n geq N$ implies
          $$mu(E^c cap geqvarepsilon) = 0$$
          What does this imply for $mu(|f_n-f|geq varepsilon)?$







          share|cite|improve this answer












          share|cite|improve this answer



          share|cite|improve this answer










          answered 8 hours ago









          Brian MoehringBrian Moehring

          1,6192 silver badges9 bronze badges




          1,6192 silver badges9 bronze badges














          • $begingroup$
            How to conclude that $mid f_n-fmid subseteq E$?
            $endgroup$
            – Danmat
            7 hours ago










          • $begingroup$
            @Danmat Forget about $E$ for a second. On $E^c,$ the functions converge uniformly. This means that there will be some $N$ such that $n geq N$ implies $|f_n - f| < varepsilon.$ In particular, on $E^c,$ $$n geq N implies geq varepsilon = emptyset.$$ This directly means $$E^c cap geq varepsilon = emptyset.$$ If you want, then you can use the property of sets that $$A^c cap B = emptyset iff B subseteq A$$
            $endgroup$
            – Brian Moehring
            7 hours ago











          • $begingroup$
            Very nice! Now, it's all clear to me!
            $endgroup$
            – Danmat
            7 hours ago
















          • $begingroup$
            How to conclude that $mid f_n-fmid subseteq E$?
            $endgroup$
            – Danmat
            7 hours ago










          • $begingroup$
            @Danmat Forget about $E$ for a second. On $E^c,$ the functions converge uniformly. This means that there will be some $N$ such that $n geq N$ implies $|f_n - f| < varepsilon.$ In particular, on $E^c,$ $$n geq N implies geq varepsilon = emptyset.$$ This directly means $$E^c cap geq varepsilon = emptyset.$$ If you want, then you can use the property of sets that $$A^c cap B = emptyset iff B subseteq A$$
            $endgroup$
            – Brian Moehring
            7 hours ago











          • $begingroup$
            Very nice! Now, it's all clear to me!
            $endgroup$
            – Danmat
            7 hours ago















          $begingroup$
          How to conclude that $mid f_n-fmid subseteq E$?
          $endgroup$
          – Danmat
          7 hours ago




          $begingroup$
          How to conclude that $mid f_n-fmid subseteq E$?
          $endgroup$
          – Danmat
          7 hours ago












          $begingroup$
          @Danmat Forget about $E$ for a second. On $E^c,$ the functions converge uniformly. This means that there will be some $N$ such that $n geq N$ implies $|f_n - f| < varepsilon.$ In particular, on $E^c,$ $$n geq N implies geq varepsilon = emptyset.$$ This directly means $$E^c cap geq varepsilon = emptyset.$$ If you want, then you can use the property of sets that $$A^c cap B = emptyset iff B subseteq A$$
          $endgroup$
          – Brian Moehring
          7 hours ago





          $begingroup$
          @Danmat Forget about $E$ for a second. On $E^c,$ the functions converge uniformly. This means that there will be some $N$ such that $n geq N$ implies $|f_n - f| < varepsilon.$ In particular, on $E^c,$ $$n geq N implies geq varepsilon = emptyset.$$ This directly means $$E^c cap geq varepsilon = emptyset.$$ If you want, then you can use the property of sets that $$A^c cap B = emptyset iff B subseteq A$$
          $endgroup$
          – Brian Moehring
          7 hours ago













          $begingroup$
          Very nice! Now, it's all clear to me!
          $endgroup$
          – Danmat
          7 hours ago




          $begingroup$
          Very nice! Now, it's all clear to me!
          $endgroup$
          – Danmat
          7 hours ago

















          draft saved

          draft discarded
















































          Thanks for contributing an answer to Mathematics Stack Exchange!


          • Please be sure to answer the question. Provide details and share your research!

          But avoid


          • Asking for help, clarification, or responding to other answers.

          • Making statements based on opinion; back them up with references or personal experience.

          Use MathJax to format equations. MathJax reference.


          To learn more, see our tips on writing great answers.




          draft saved


          draft discarded














          StackExchange.ready(
          function ()
          StackExchange.openid.initPostLogin('.new-post-login', 'https%3a%2f%2fmath.stackexchange.com%2fquestions%2f3332265%2falmost-uniform-convergence-implies-convergence-in-measure%23new-answer', 'question_page');

          );

          Post as a guest















          Required, but never shown





















































          Required, but never shown














          Required, but never shown












          Required, but never shown







          Required, but never shown

































          Required, but never shown














          Required, but never shown












          Required, but never shown







          Required, but never shown







          Popular posts from this blog

          ParseJSON using SSJSUsing AMPscript with SSJS ActivitiesHow to resubscribe a user in Marketing cloud using SSJS?Pulling Subscriber Status from Lists using SSJSRetrieving Emails using SSJSProblem in updating DE using SSJSUsing SSJS to send single email in Marketing CloudError adding EmailSendDefinition using SSJS

          Кампала Садржај Географија Географија Историја Становништво Привреда Партнерски градови Референце Спољашње везе Мени за навигацију0°11′ СГШ; 32°20′ ИГД / 0.18° СГШ; 32.34° ИГД / 0.18; 32.340°11′ СГШ; 32°20′ ИГД / 0.18° СГШ; 32.34° ИГД / 0.18; 32.34МедијиПодациЗванични веб-сајту

          19. јануар Садржај Догађаји Рођења Смрти Празници и дани сећања Види још Референце Мени за навигацијуу